Đến nội dung

Stranger411 nội dung

Có 85 mục bởi Stranger411 (Tìm giới hạn từ 30-04-2020)



Sắp theo                Sắp xếp  

#345861 Ảnh thành viên

Đã gửi bởi Stranger411 on 11-08-2012 - 14:58 trong Góc giao lưu

Hình đã gửi
Up cái này phát!
Ông quả nhìn to quá!

Mấy ông ơi !!
cái bảng đó bây h ai giữ thế ;p
Bữa nào bán đấu giá trên Vmf đi :D



#431535 Topic về tổ hợp, các bài toán về tổ hợp

Đã gửi bởi Stranger411 on 29-06-2013 - 11:02 trong Tổ hợp và rời rạc

Bài 2: (PP ánh xạ, chứng minh đặc tính tổ hợp) Gọi an là số các xâu nhị phân độ dài n không chứa chuỗi con 010, bn là số các xâu nhị phân độ dài n không chứa chuỗi con 0011 hoặc 1100. Chứng minh rằng bn+1 = 2an với mọi n nguyên dương. 

Giải bài 2:
Xét một xâu nhị phân bất kì  $\left \{ x_1, x_2,...,x_n \right \}$.
Ta xây dựng một xâu nhị phân $\left \{ y_1, y_2,...,y_{n+1} \right \}$ sao cho $y_1 =0$ và $y_k = x_1 + x_2 +...+ x_k(mod2)$
Rõ ràng, xâu $\left \{ x_1, x_2,...,x_n \right \}$ có dạng $a_n$ khi và chỉ khi $\left \{ y_1, y_2,...,y_{n+1} \right \}$ có dạng $b_n$. Nói cách khác đó là một song ánh biến các xâu có dạng $a_n$ thành các xâu có dạng $b_{n+1}$ bắt đầu bằng $0$.
Với mỗi xâu $\left \{ y_1, y_2,...,y_{n+1} \right \}$, ta thay các kí tự 1 bởi 0 và 0 bởi 1, ta được một xâu khác cũng có dạng $b_{n+1}$ nhưng bắt đầu bởi $1$.
Từ đó cho ta: $b_{n+1} = 2a_n$.
 

Bài 3 (Toán trò chơi-Nguồn: Tournament of the Towns). Trên bảng ô vuông $20$x$20$ mỗi ô có $1$ quân cờ. $A$ chọn một số thực $d$ và $B$ phải đưa mỗi quân cờ đi tới ô cách ô ban đầu nó đứng một khoảng cách ít nhất là $d$ (khoảng cách giữa $2$ ô được tính theo khoảng cách tâm của $2$ ô đó) và mỗi ô chỉ có thể có $1$ quân cờ. Hỏi với điều kiện gì của $d$ thì $B$ có thể thao tác thoả mãn điều kiện trên?

Hỏi tương tự với bảng $21$x$21$

Với bài này có lẽ ta phải chia bàn cờ ra 4 phần.
Cái đáp số hơi lằng nhằng, không biết có đúng không.




#431947 Topic về tổ hợp, các bài toán về tổ hợp

Đã gửi bởi Stranger411 on 30-06-2013 - 22:38 trong Tổ hợp và rời rạc

Bài 11:(Lý thuyết đồ thị, APMO 1989) Chứng minh rằng một đồ thị n đỉnh, k cạnh thì sẽ có ít nhất $\frac{{k\left( {4k - {n^2}} \right)}}{{3n}}$ tam giác.

Câu này được chế biến lại làm đề chọn đội tuyển của KHTN năm nào đó thì phải, nhưng cũng chỉ là điều kiện cần.
Giải bài 11:
Gọi $D_i$ là bậc của đỉnh $i$ nào đó.
Nếu 2 đỉnh $i$ và $j$ liên thông thì tồn tại ít nhất $D_i + D_j - 2$ cạnh khác nổi đến $n-2$ đỉnh khác của đồ thị.
Nên có $D_i + D_j - n$ tam giác có đỉnh là $j$ và $j$. Vậy số tam giác có chứa cạnh $ij$ được tạo thành phải tối thiểu là:
$\sum_{i,j} {\frac{D_i + D_j -n}{3}}=\sum_{i,j} {\frac{D_i + D_j}{3} - \frac{nk}{3}} = \sum_{i} {\frac{D_i^2}{3} - \frac{nk}{3}}$

Vậy số tam giác tối thiểu được tạo thành là:

$\sum_{i}{\frac{D_i^2}{3}} -\frac{nk}{3} \geq \frac{1}{3n}\left ( \sum_{i}{D_i} \right )^2 - \frac{nk}{3} = \frac{4k^2}{3n} -\frac{nk}{3}$
Q.E.D.

 

Bài 6: (truy hồi/đa thức,giải tích tổ hợp,PTNK 2009) Cho số nguyên dương n. Có bao nhiêu số chia hết cho 3, có n chữ số và các chữ số đều thuộc {3, 4, 5, 6}?

Sau đây là cách giải PP truy hồi:

Gọi $a_n$ là số các số có $n$ chữ số lập từ ${3, 4, 5, 6}$ và chia hết cho 3, còn $b_n$ là số các số có $n$ chữ số lập từ ${3, 4, 5, 6}$ và không chia hết cho 3. Khi đó ta có

$a_n = 2a_{n-1} + b_{n-1}(1)$

$b_n = 2a_{n-1} + 3b_{n-1}(2)$

Từ (1) suy ra $b_{n-1} = a_n – 2a_{n-1}$, thay n à n+1 thì được $b_n = a_{n+1} – 2a_n$. Thay vào (2), ta được

$a_{n+1} – 2a_n = 2a_{n-1} + 3(a_n – 2a_{n-1})$

$a_{n+1} – 5a_n + 4a_{n-1} = 0$.

Giải phương trình sai phân này, với chú ý rằng a1 = 2, a2 = 6, ta tìm được

\[{a_n} = \frac{{{4^n} + 2}}{3}.\]

Với cách đặt trên ta có thể có công thức truy hồi khác là $b_n = 4^n - a_n$
từ đó suy ra $a_{n+1} = 2a_{n} + b_{n}= a_n + 4^n$
Nên $a_{n} = 4^{n-1} + 4^{n-2} +...+ a_{1} = \frac{4^n + 2}{3}$




#432471 Topic về tổ hợp, các bài toán về tổ hợp

Đã gửi bởi Stranger411 on 03-07-2013 - 10:00 trong Tổ hợp và rời rạc

Bài 17 (cực trị-bất đẳng thức tổ hợp)

Cho tập hợp $X=\left \{ 1,2,...,50 \right \}$. Tìm số nguyên dương nhỏ nhất $k$ sao cho mọi tập con gồm $k$ phần tử của $X$ đều chứa hai phần tử phân biệt $a$ và $b$ sao cho $ab$ chia hết cho $a+b$

Ta có các cặp số thoả mãn $ab$ chia hết $a+b$: $(3;6);(4;12);(5;20);(6;12);(6;30);(7;42);(8;2);(9;18);(10;15);(10;40);(12;24);(12;36);(14;35);(15;30);(16;48);(18;36);(20;30);(21;28);(21;42);(24;40);(24;48);(30;45);(36;45)$

Tổng hợp các số lại tập $A$, ta có:\[A = \left\{ {3;4;5;6;7;8;9;10;12;14;15;16;18;20;21;24;30;35;36;40;45;48} \right\}\]

\[ \Rightarrow k \ge \left| {X\backslash A} \right| + \left[ {\frac{{\left| A \right|}}{2}} \right] + 1 = 39\]

Vậy $min(k)=39$

--------------------------------------------------------------------------------------------------------------------------------

Đây là kết quả sau 1h bấm máy tính của mình, các bạn có cách nào hay hơn thì đưa lên để mọi người cùng tham khảo

Giải bài 17:
Chứng minh $k \ge 39$
Các cặp số thoả mãn $ab$ chia hết $a+b$: $(3;6);(4;12);(5;20);(6;12);(6;30);(7;42);(8;2);(9;18);(10;15);(10;40);(12;24);(12;36);(14;35);(15;30);(16;48);(18;36);(20;30);(21;28);(21;42);(24;40);(24;48);(30;45);(36;45)$

Xét tập $M=(6,12,15,18,20,21,24,35,40,42,45,48)$. Vì 23 cặp trên đều có phần tử thuộc $M$ nên tập $X\M$ không thỏa mãn bài toán. Mà $|X\M|=38 \rightarrow k \ge 39$

Chứng minh mọi tập có 39 phần tử đều thỏa mãn bài toán:
Xét tập $A$ bất kì gồm 39 phần tử của $X$.
Chọn 12 cặp số trong 23 cặp trên sao cho các phần tử không trùng nhau: $(3;6);(4;12);(5;20);(7;42);(8;2);(9;18);(10;15);(14;35);(18;36);(21;28);(24;40);(30;45)$

12 cặp trên chứa 24 phần tử của $X$. Nên $X$ chỉ còn lại 26 phần tử.
Vậy ít nhất 13 phần tử của $A$ của phải thuộc 12 cặp trên.
Theo nguyên lí drichlet thì $A$ chứa ít nhất 1 trong 12 cặp trên.
Từ đó tập $A$ thỏa mãn đề bài.




#432263 Topic về tổ hợp, các bài toán về tổ hợp

Đã gửi bởi Stranger411 on 02-07-2013 - 12:05 trong Tổ hợp và rời rạc

Post tiếp 2 bài nữa làm ... điểm tâm  :wacko: :

Bài 15:(APMO 1998) Cho F là tập tất cả các bộ (A1, . . . ,An) sao cho mỗi Ai là một tập con của {1, 2, . . . , 1998}. Ký hiệu |A| là số các phần tử của tập hợp A. Hãy tính

$\sum\limits_{({A_1},{A_2},...,{A_n}) \in F} {|{A_1} \cup {A_2} \cup ... \cup {A_n}|} $

Bài 16:(Bulgaria 1996) HÌnh chữ nhật $m \times n ( m,n \in N, m,n>1)$ được chia thành $mn$ hình vuông đơn vị. Có bao nhiêu cách xoá 2 hình vuông sao cho phần còn lại có thể lát kín bởi các domino?

Tại sao các bạn cứ thích post những bài không có lời giải ở trong sách ra thế nhỉ :))
Mình nhớ không nhầm thì mấy bài này trong chuyên đề của thầy Huỳnh Tấn Châu để học sinh tự giải. Sau đây là cách của mình, bạn có cách khác thì post lên cho mọi người tham khảo.
Giải bài 15:

Bài này đếm bằng truy hồi.
Tập $(1, 2, . . . , 1998)$ có tất cả $2^{1998}$ tập con $A_i$ nên tổng cần tính có tất cả $2^{1998n}$ bộ $n$ số.
Bước 1: Với các tập con $(A_1, A_2 ... , A_n)$ của $(1, 2, . . . , 1997)$, ta có thể thêm hoặc không thêm vào mỗi tập $A_i$ phần tử $1998$ để tạo thành tập con của tập $(1, 2, . . . , 1998)$. Vậy từ bộ $(A_1, A_2 ... , A_n)$ ta có $2^n$ bộ gồm các tập con của $\left \{1, 2, . . . , 1998 \right \}$.

Bước 2: Bây h chỉ còn xét các tập con $(A_1, A_2 ... , A_n)$ của các bộ còn lại. Làm tương tự như trên thì ta có được $(2^n -1).2^{n(m-1)}$.
Tình tổng lại, ta có:

$\sum\limits_{({A_1},{A_2},...,{A_n}) \in (1,...,1998)} {|{A_1} \cup {A_2} \cup ... \cup {A_n}|} = (2^n.\sum\limits_{({A_1},{A_2},...,{A_n}) \in (1,...,1998)} {|{A_1} \cup {A_2} \cup ... \cup {A_n}|}) + (2^n -1) 2^{n(m-1)}$
Từ đó, ta có: $\sum\limits_{({A_1},{A_2},...,{A_n}) \in F} {|{A_1} \cup {A_2} \cup ... \cup {A_n}|} = 1998(2^{1998n} - 2^{1997n})$.

 

Mở rộng bài 15: Tính:
$P= \sum_{(A_{1},A_{2},...,A_{k})\in F_{k}}\sum_{b\in (A_{1}\cup A_{2}\cup ...\cup A_{k})}b$
$S= \sum_{(A_{1},A_{2},...,A_{k})\in F_{k}}\sum_{i=1}^{k}\sum_{b\in A_{i}}b$
 

Bài 7: (Đề đề nghị 30/4 -2012) Trong một kì thi hoa hậu, mỗi thành viên của ban giám khảo được quyền đề xuất $10$ người đẹp vào vòng chung kết. Một nhóm người đẹp được gọi là nhóm ưng ý đối với giám khảo A nếu trong nhóm đó có ít nhất một người đẹp mà A đề xuất. Biết rằng với 6 giám khảo bất kỳ luôn tồn tại một nhóm gồm 2 người đẹp là nhóm ưng ý đối với mỗi giám khảo trong 6 giám khảo đó. Chứng minh rằng tồn tại một nhóm gồm 10 người đẹp là nhóm ưng ý đối với mỗi thành viên của ban giám khảo.

Đề này của trường chuyên Nguyễn Tất Thành - Komtum và họ không đưa đáp án nên trong sách không có lời giải.
Mình nghĩ bạn Ispectorgadget cũng không có đáp án cho bài này. (Trừ khi bạn ấy học trường chuyên NTT)

Bài này chắc không thể giải bằng đồ thị vì phải xét tới 2 đối tượng là hoa hậu và giám khảo.
Thôi thì phát biểu lại dưới dạng tập hợp để mọi người cùng nghiên cứu:
Cho $X=\{1;2;...;n\}$. Và $A_1 ,A_2, ..., A_m$ là các tập con của $X$.
Biết 6 phần tử bất kì của $X$ luôn thuộc $|A_i\cup A_j|$ nào đó.
Chứng minh tồn tại $i_1;i_2;..;i_{10}$ phân biệt mà $|\bigcup\limits_{j=1}^{10}A_{i_j}|=X$.




#346389 Hội những người độc thân thích chém gió !

Đã gửi bởi Stranger411 on 13-08-2012 - 09:42 trong Góc giao lưu

Duyệt cho anh =))

Ố ố :-ss
Mình là con gái mà ;)
Anh nào đâu ;))



#312453 Topic: INEQUALITIES (PART II)

Đã gửi bởi Stranger411 on 24-04-2012 - 19:32 trong Bất đẳng thức - Cực trị

Problem 8: Cho a,b,c là các số thực thỏa $a+b+c=3$. Chứng minh rằng:
$$\frac{a^2-bc}{a^2+3}+\frac{b^2-ac}{b^2+3}+\frac{c^2-ab}{c^2+3}\geq 0$$

Bài này khá yếu.
Bằng cách phân tích trực tiếp, ta được:
$$\left( {{a}^{2}}+{{b}^{2}}+{{c}^{2}}-ab-bc-ca \right)\sum{\frac{{{\left( a-b \right)}^{2}}}{\left( {{a}^{2}}+3 \right)\left( {{b}^{2}}+3 \right)}}\ge 0$$
Ta có đpcm. $\blacksquare$

Problem 9: Cho a,b,c là các số thực thỏa $a+b+c=1$. Chứng minh rằng:
$$\frac{\left( {{a}^{2}}+{{b}^{2}} \right)}{{{\left( a+b \right)}^{2}}}\frac{\left( {{c}^{2}}+{{b}^{2}} \right)}{{{\left( c+b \right)}^{2}}}\frac{\left( {{a}^{2}}+{{c}^{2}} \right)}{{{\left( a+c \right)}^{2}}}\ge \frac{3}{8}\left( {{a}^{2}}+{{b}^{2}}+{{c}^{2}} \right)$$



#430322 Đề thi vào lớp 10 THPT chuyên Lê Quí Đôn Đà Nẵng 2013-2014 (Hệ số 2)

Đã gửi bởi Stranger411 on 24-06-2013 - 20:42 trong Tài liệu - Đề thi

 

Bài 4. (1,5 điểm)

          Cho một tháp số (gồm 20 ô vuông giống nhau) như hình vẽ. Mỗi ô vuông được ghi một số nguyên dương n với $1\leq n\leq 20$, hai ô vuông bất kỳ không được ghi cùng một số. Ta quy định trong tháp số này 2 ô vuông kề nhau là 2 ô vuông có chung cạnh. Hỏi có thể có cách ghi nào thỏa mãn điều kiện: Chọn 1 ô vuông bất kỳ (khác với các ô vuông được đặt tên a, b, c, d, e, f, g, h như hình vẽ) thì tổng của số được ghi trong ô đó và các số được ghi trong 3 ô vuông kề với nó chia hết cho 4 ?

1001458_10200372004477893_499076836_n.jp




#441489 IMO shortlist 2012 - Problems + Solution

Đã gửi bởi Stranger411 on 09-08-2013 - 15:20 trong Thi HSG Quốc gia và Quốc tế

Mới có bản Scan thôi, các bạn dùng tạm :)

File gửi kèm




#368262 $\sum_{k=0}^{\left\lfloor\frac{n...

Đã gửi bởi Stranger411 on 09-11-2012 - 21:48 trong Tổ hợp và rời rạc

Dạo này box tổ hợp và rời rạc của VMF có vẻ trầm lắng!
Để tránh tình trạng này kéo dài, tôi xin khuấy động bằng một bài nho nhỏ

Cho số nguyên $n\ge 3$. Chứng minh đẳng thức:

$\sum_{k=0}^{\left\lfloor\frac{n}{3}\right\rfloor} \binom{n}{3k} = \dfrac{2^n+(-1)^n\left(3\left\lfloor\frac{n}{3}\right\rfloor-3\left\lfloor\frac{n-1}{3}\right\rfloor-1\right)}{3}$

Chào các anh. Bài này dùng hàm sinh kết hợp với RUF tức là Root of Unity Filter đó ạ :D

Định lí: Cho số nguyên dương $n$. Đặt $k=cos \frac{2 \pi}{n}+i sin \frac{2 \pi}{n}$.
Xét đa thức $f(x)=a_n x^n+a_{n-1} x^{n-1}+...+a_1 x+a_0$. Khi đó:
$a_0 + a_{n} + a_{2n} + ... = \frac{1}{n}[f(1)+f(k)+f(k^2)+...+f(k^{n-1})]$

Bài toán: Đặt $k=cos \frac{2 \pi}{3}+i .sin \frac{2 \pi}{3}$.
Áp dụng định lí trên, ta được:
$\sum_{k=0}^{\left\lfloor\frac{n}{3}\right\rfloor} \binom{n}{3k} = \frac{(1+k)^n + (1+ k^2 )^n + (1+1)^n}{3}$
$=\frac{(-1)^n(k^n + k^{2n}) + 2^n}{3}$
từ đó, ta có kết quả bài toán.

Remark: Ngoài ra, ta còn kết quả khác:
Nếu $n$ chia hết cho 3: $\sum_{k=0}^{\infty } \binom{n}{3k} = \frac{2^n + 2(-1)^n}{3}$
Nếu $n$ không chia hết cho 3: $\sum_{k=0}^{\infty } \binom{n}{3k} = \frac{2^n + (-1)^{n+1}}{3} $


ps hxthanh@: thầy cho em hỏi ạ :D hồi rất lâu rồi ấy,thầy có lập 1 topic tên là "Dãy số đầu năm" trong box Đại số THCS. Sau này mod chuyển đi đâu mất :( Thầy cho em cái link để tham khảo được không ạ, Hồi đó em còn non nên chưa đọc được nhiều :D



#422841 Chứng minh tiếp tuyến

Đã gửi bởi Stranger411 on 01-06-2013 - 16:36 trong Hình học

Bài toán khá hay đó bạn:

Qua $M$ vẽ đường thẳng song song với $d$,Cắt $(O)$ tại $N$,cắt $OH$ tại $I$.

 

Vì $HA=HB$ và $MN//AB$ nên $M(AB,HN)=-1$,Chiếu chùm điều hòa lên $(O)$ ta có tứ giác $DCEN$

 

Kí hiệu mà bạn dùng chỗ nào là thế nào vậy ?
mình xem lại lí thuyết về chùm điều hòa mà không thấy.




#422825 Chứng minh tiếp tuyến

Đã gửi bởi Stranger411 on 01-06-2013 - 15:17 trong Hình học

Cho đường tròn $(O)$ và đường thẳng $d$. Gọi $H$ là hình chiếu của $O$ lên $d$. Cho 2 điểm $A,B \in d$ sao cho $HA =HB$. Lấy $M$ bất kì trên $(O)$. $MH,MA,MB$ lần lượt cắt $(O)$ tại $C,D,E$. Gọi $S$ là giao điểm của d và $DE$. Chứng minh $SC$ là tiếp tuyến của đường tròn $(O)$.




#422851 Chứng minh tiếp tuyến

Đã gửi bởi Stranger411 on 01-06-2013 - 17:28 trong Hình học

Qua $M$ vẽ đường thẳng song song với $d$,Cắt $(O)$ tại $N$,cắt $OH$ tại $I$.

 

Vì $HA=HB$ và $MN//AB$ nên $M(AB,HN)=-1$

Theo mình thấy chỗ này nên chứng minh lại bổ đề sau:

 

Bổ đề 1: Cho $a,b,c,d$ là chùm đường thẳng tâm $O$. Đường thẳng $\delta$ không đi qua $O$, cắt $a,b,c,d$ tại $A,B,C,D$. Đường thẳng $\delta'$ không đi qua $O$, cắt $a,b,c$ tại $A',B',C'$. Khi đó $\delta' // d$ thì $(ABCD)=(A'B'C')$.
 

Áp dụng bổ đề trên thì $M(ABHN)= (ABH) = -1$.




#385774 VMO 2013 - Bài 1. Hệ phương trình

Đã gửi bởi Stranger411 on 11-01-2013 - 23:00 trong Phương trình - Hệ phương trình - Bất phương trình

Cách khác: (không dùng Minkowski :D)
ĐKXĐ: $x\neq \dfrac{m\pi }{2}$, $y\neq \dfrac{n\pi }{2}$ ($m$, $n\in \mathbb{Z}$), $xy\geq 0$, $x+y\neq 0$.
Ta chứng minh:
$(\sqrt{{{\sin }^{2}}x+\frac{1}{{{\sin }^{2}}x}}+\sqrt{{{\cos }^{2}}y+\frac{1}{{{\cos }^{2}}y}})^2 + (\sqrt{{{\sin }^{2}}y+\frac{1}{{{\sin }^{2}}y}}+\sqrt{{{\cos }^{2}}x+\frac{1}{{{\cos }^{2}}x}})^2 \ge 20$

Áp dụng $\frac{1}{\sin ^2 x} + \frac{1}{\cos ^2 x} \ge 4$ $ \forall x \neq 0$, ta có:
$(\sqrt{{{\sin }^{2}}x+\frac{1}{{{\sin }^{2}}x}}+\sqrt{{{\cos }^{2}}y+\frac{1}{{{\cos }^{2}}y}})^2 + (\sqrt{{{\sin }^{2}}y+\frac{1}{{{\sin }^{2}}y}}+\sqrt{{{\cos }^{2}}x+\frac{1}{{{\cos }^{2}}x}})^2 $
$\ge 10 + 2 \sqrt{{{\sin }^{2}}x+\frac{1}{{{\sin }^{2}}x}}.\sqrt{{{\cos }^{2}}y+\frac{1}{{{\cos }^{2}}y}} + 2 \sqrt{{{\sin }^{2}}y+\frac{1}{{{\sin }^{2}}y}}.\sqrt{{{\cos }^{2}}x+\frac{1}{{{\cos }^{2}}x}}$

Vậy chỉ cần chứng minh:
$ \sqrt{{{\sin }^{2}}x+\frac{1}{{{\sin }^{2}}x}}.\sqrt{{{\cos }^{2}}y+\frac{1}{{{\cos }^{2}}y}} + \sqrt{{{\sin }^{2}}y+\frac{1}{{{\sin }^{2}}y}}.\sqrt{{{\cos }^{2}}x+\frac{1}{{{\cos }^{2}}x}} \ge 5$ $ \forall x \neq 0$
từ đó quay về chứng minh:
$ \left(\sin^2x + \dfrac{1}{\sin^2x}\right)\left(\cos^2x + \dfrac{1}{\cos^2x}\right) \geq \left(\dfrac{5}{2}\right)^2.$và $ \left(\sin^2y + \dfrac{1}{\sin^2y}\right)\left(\cos^2y + \dfrac{1}{\cos^2y}\right) \geq \left(\dfrac{5}{2}\right)^2.$

Sử dụng bất đẳng thức Cauchy-Schwarz và AM-GM ta có
$$ \begin{aligned} \left(\sin^2x + \dfrac{1}{\sin^2x}\right)\left(\cos^2x + \dfrac{1}{\cos^2x}\right) \ & \geq \left(|\sin x\cos x|+\dfrac{1}{|\sin x\cos x|}\right)^2 \\ & =\left(\dfrac{|\sin 2x|}{2}+\dfrac{1}{2|\sin 2x|}+\dfrac{3}{2|\sin 2x|}\right)^2 \\ & \geq \left( 1+\dfrac{3}{2}\right)^2=\left(\dfrac{5}{2}\right)^2.\end{aligned}$$
Ta có (đpcm).
Suy ra: $\tan x = \tan y= \pm 1$ và $x=y$
Từ đó, ta được $ x=y= \dfrac{\pi }{4}+\dfrac{k\pi }{2}$ ($k\in \mathbb{Z}$).



#441245 Mở rộng Problem 4 - IMO 2013

Đã gửi bởi Stranger411 on 08-08-2013 - 14:40 trong Hình học

1 bài mở rộng của thầy Quang Hùng trong GGTH lần 5

Cho tam giác $ABC$, đường tròn $(I)$ đi qua $B,C$ cắt CA,AB tại $N,M$ khác $B,C$.
Đặt $H = BN \cap CM$. Gọi $d$ là đường thẳng qua $I$ vuông góc với $AH$. Lấy $W$ bất kì trên $d$.
$WK,WL$ là đường kính của đường tròn ngoại tiếp các tam giác $WBM,WCN$.
Chứng minh $K,H,L$ thẳng hàng.
 

TBr.png




#438572 Chứng minh ba đường tròn đồng quy

Đã gửi bởi Stranger411 on 27-07-2013 - 13:22 trong Hình học

Cho tứ giác toàn phần $ABCDEF$. Chứng minh các đường tròn đướng kính $AC,BD,EF$ đồng quy tại 2 điểm.

NhaSinhvien.net-bai1.png




#346020 Tìm các số nguyên dương $a,b,c$ sao cho $\frac{a^{2}+b^{2...

Đã gửi bởi Stranger411 on 11-08-2012 - 23:47 trong Số học

He he áp dụng cái bổ đề anh Tường nói thì bài này làm ngon
Giải như sau:
Bổ đề: $p \in \mathbb{P}, p \equiv 2 \pmod{3}, a^2+3b^2 \vdots p \Leftrightarrow p|a,b$

Em Tạ giải kinh quá :P
Chắc thằng Tường nó chả bao giờ theo kịp đâu :P

Cách khác:
Đẳng thức được viết lại như sau $(a+b+c)^2=(3k+2)(ab+bc+ca)$
Chọn số nguyên tố $p$ sao cho $\left\{ \begin{gathered}
{p^{2a - 1}}|3k + 2 \\
{p^{2a}}|3k + 2 \\
\end{gathered} \right.$
$ \Rightarrow \left\{ \begin{gathered}
{p^a}|a + b + c \\
p|ab + bc + ca \\
\end{gathered} \right.$
$c \equiv - a - b(\bmod p) \Rightarrow p|a^2 + ab + b^2 \Rightarrow p|{(2a + b)^2} + 3{b^2}$
$ \Rightarrow \left( {\frac{{ - 3}}{p}} \right) = 1$. Và điều này vô lí vì $p \equiv 2(\bmod 3)$.
Vậy không tồn tại $a,b,c$ thỏa mãn bài toán. $\blacksquare$



#346392 Tìm các số nguyên dương $a,b,c$ sao cho $\frac{a^{2}+b^{2...

Đã gửi bởi Stranger411 on 13-08-2012 - 09:49 trong Số học

muốn có -3 là thặng dư toàn phương của p thì 1 trong 2 số 2a+b hoặc b không chia hết cho p,nên theo nguyenta a,b,c chia hết cho p là hợp lí

Gì nữa đây bạn :P
$\left( {\frac{{ - 3}}{p}} \right) = 1$ là kí hiệu Lengdre ;) Chớ có phải thăng dư toàn phuơng gì đâu ;))
Cách em Nguyenta98 là lùi vô hạn. Cách mình là dùng các định lí về thăng dư bậc 2 thôi ;)



#346426 Tìm các số nguyên dương $a,b,c$ sao cho $\frac{a^{2}+b^{2...

Đã gửi bởi Stranger411 on 13-08-2012 - 12:03 trong Số học

ý m` là không có cách của bạn đâu,nếu 2a+b chia hết cho p thì làm quoái j` mà vô lí chứ,cũng giống như a mũ 2 + b mũ 2 có ước nguyên tố dạng thì 4p+3 thì a,b cùng chia hết cho SNT đó,làm j` có chuyện vô lí ở đâu,khi sử dụng kí hiêu lengdre ta đã ngầm hiểu tử số của nó không chia hết cho mẫu r` :lol:

Trước khi nói cái gì thì nên coi lại kiến thức của mình một tí đi nhá ;)

Định nghĩa về kí hiệu Lengdre:
Cho số nguyên tố $p$ và số nguyên $a$. Khi đó, ta có:
+$\left( {\frac{{ a}}{p}} \right) = 1$ nếu $a \not \vdots p$ và $a$ là số chính phương $mod(p)$
+$\left( {\frac{{ a}}{p}} \right) = -1$ nếu $a \not \vdots p$ và $a$ không là số chính phương $mod(p)$
+$\left( {\frac{{ a}}{p}} \right) = 0$ nếu $a \vdots p$

Trờ lại bài toán:
Vì vậy nếu $\left( {\frac{{ - 3}}{p}} \right) = 1$ thì hoàn toàn vô lí vì ta chọn $p \equiv 2(\bmod 3)$



#347148 Tìm các số nguyên dương $a,b,c$ sao cho $\frac{a^{2}+b^{2...

Đã gửi bởi Stranger411 on 16-08-2012 - 10:37 trong Số học

sai từ chỗ này và nguyên nhân là do làm tắt $p|{(2a + b)^2} + 3{b^2}$
$ \Rightarrow \left( {\frac{{ - 3}}{p}} \right) = 1$
muốn dùng lengdre(hay tiếng việ gọi là thặng dư toàn phương) trước tiên ta phải đưa nó về dạng (mà ở đây) là
a2$\equiv$-3 (mod p) cái đã,mà ở đây muốn đưa về dạng này ta phải giả sử a không chia hết cho p,''vậy nên thiếu TH a,b chia hết cho p'',mà TH này luôn đúng,nếu không thấy dc thì cho a=b=p ta có 12p2 chia hết cho p ,vì vậy có giải kiểu gì đi nữa vẫn phải thông qua a,b,c chia hết cho p rồi mới giải tiếp,nên không có cách bạn stranger nói

Nói chuyện vs Uyenha cực kì bực mình @@!
Mình và mọi người đã ko muốn nói rồi mà bạn cứ thích cãi cùn.

Trước đó, MOD đã gộp bớt vài bài của bạn để tránh spam trong topic.
Thắc mắc thì ko phải là tội nhưng cứ nói dai như thế người ta chả thích tí nào đâu bạn :)

Mời bạn tham khảo thêm về kí hiệu Lengdre:
File gửi kèm  Cong Thuc Legendre.pdf   67.83K   1682 Số lần tải



#346577 Tìm các số nguyên dương $a,b,c$ sao cho $\frac{a^{2}+b^{2...

Đã gửi bởi Stranger411 on 13-08-2012 - 21:29 trong Số học

$\Rightarrow 2a^2+2ab+2b^2 \vdots p \Rightarrow 4a^2+4ab+4b^2 \vdots p \Rightarrow (2a+b)^2+3b^2 \vdots p$

$c \equiv - a - b(\bmod p) \Rightarrow p|a^2 + ab + b^2 \Rightarrow p|{(2a + b)^2} + 3{b^2}$
$ \Rightarrow \left( {\frac{{ - 3}}{p}} \right) = 1$. Và điều này vô lí vì $p \equiv 2(\bmod 3)$.
Vậy không tồn tại $a,b,c$ thỏa mãn bài toán. $\blacksquare$

Bạn xem lại @@!
Bài mình và bài Tạ để đi đến $p|{(2a + b)^2} + 3{b^2}$ để suy ra vô lí mà (:|

Bạn nói lại xem mình sai chỗ nào (:|



#313293 Balkan MO 2012

Đã gửi bởi Stranger411 on 29-04-2012 - 14:46 trong Thi HSG Quốc gia và Quốc tế

Balkan MO 2012 - 28 April 2012

Bài 2. Prove that \[ \sum_{cyc}(x+y)\sqrt{(z+x)(z+y)}\geq 4(xy+yz+zx), \] for all positive real numbers $x,y$ and $z$.

Tuy là đề thi quốc gia nhưng mình thấy bài này khá lỏng :D
Cách 1:
Áp dụng bđt Cauchy-Schwarz, ta có:


$\left( x+y \right)\sqrt{\left( z+x \right)\left( z+y \right)}\ge \left( x+y \right)\left( z+\sqrt{xy} \right)$

$=\left( x+y \right)z+\left( x+y \right)\sqrt{xy}\ge \left( x+y \right)z+2xy$
Cách 2: Bằng cách dùng bổ đề của huymit_95, ta viết lại bđt như sau:
$\sum {\frac{1}{{\sqrt {\left( {x + y} \right)\left( {x + z} \right)} }}} \ge \frac{{4\left( {xy + yz + zx} \right)}}{{\left( {x + y} \right)\left( {y + z} \right)\left( {z + x} \right)}}$
Ta có: $\left( {x + y} \right)\left( {y + z} \right)\left( {z + x} \right) \ge \frac{8}{9}\left( {xy + yz + zx} \right)\left( {x + y + z} \right)$
Nên ta cần chứng minh: $\sum {\frac{1}{{\sqrt {\left( {x + y} \right)\left( {x + z} \right)} }}} \ge \frac{9}{{2\left( {x + y + z} \right)}}$
Và bđt này hoàn toàn đúng với bđt Cauchy-Schwarz:
$\sum {\frac{1}{{\sqrt {\left( {x + y} \right)\left( {x + z} \right)} }}} \ge \sum {\frac{2}{{x + y + 2z}}} \ge \frac{9}{{2\left( {x + y + z} \right)}}$



#343333 Đếm số cách lát các quân đôminô

Đã gửi bởi Stranger411 on 04-08-2012 - 13:57 trong Tổ hợp và rời rạc

Cho một hình chữ nhật có kích thước $2 \times n$ được đánh số thứ tự từ trái sang phải là $1,2,, \cdot,n$ ở hàng thứ nhất và $n+1,n+2, \cdot,2n$ ở hàng thứ 2.

Lát chúng bằng các quân đôminô $1\times2$ sao cho
1) Phủ kín hình chữ nhật và ko có 2 quân nào chồng lền nhau.
2) Vơi $n$ lẻ, ta được phép bổ sung thêm 1 quân đôminô "đặc biệt" sao cho có thể phủ kín ô $n$ và $n+1$
Đếm số cách lát thỏa mãn đề bài.



ps: Thực chất đây là bài toán tập hợp liên quan đến các phần tử tốt.



#346443 Các bài toán về số Ferma - Các bài giảng của LEUNG Tat-Wing

Đã gửi bởi Stranger411 on 13-08-2012 - 13:46 trong Tài liệu, chuyên đề, phương pháp về Số học

Mới tìm được trong đống bài giảng của ông LEUNG Tat-Wing 1 bài viết rất hay về Số Ferma.

Công nhận mấy giáo sư người Tàu rất đáng khâm phục. Mời các bạn tham khảo nó :D

File gửi kèm




#375486 $\frac{n}{m}=\sum_{k=1}^{p-...

Đã gửi bởi Stranger411 on 06-12-2012 - 00:19 trong Số học

Cho $p>3$ là một số nguyên tố.

Đặt $\begin{cases}{n\over m}=\sum_{k=1}^{p-1}{1\over k}\\ \mathrm{gcd}(n,m)=1\end{cases}$

Chứng minh rằng: $p\big| n$

Bài này có nhiều cách giải lắm thầy ạ :D
Theo em cách giải ngắn gọn nhất là sử dụng $\mathbb{Z}/p\mathbb{Z}$

Lời giải:
Ta xét bài toán trong $\mathbb{Z}/p\mathbb{Z}$, do tính tồn tại duy nhất của nghich đảo modulo, ta có:
$\overline{1}^{-1} + \overline{2}^{-1} +...+ \overline{p-1}^{-1} = \overline{1} + \overline{2} +...+ \overline{p-1} = \overline{1+2+...+ p-1} = \overline{\frac{p(p-1)}{2}} \equiv 0$
Q.E.D